Bryant - Course 6. Estate Planning. 11. Business Planning Flashcards

1
Q

Module Introduction

Estate planning strategies for businesses are critical to ensure that business continuity is properly mapped out and arrangements are made in case of the current owner’s death, disability, retirement, or withdrawal from the business. Estate freeze rules can help senior family members of a family corporation or partnership (where family members hold the majority interest) who would like to retain control of the business and pass the baton gradually to the younger generation. They may do so by retaining the preferred stock and passing on the business appreciation to the successors.

A

The Business Planning module, which should take approximately four hours to complete, will explain the estate freeze rule, valuation techniques, the business continuation process, and intra-family business transfer techniques.

Upon completion of this module, you should be able to:
* Describe the freeze rules,
* Identify valuation discounts,
* Define business continuation and its considerations, and
* Discuss intra-family business transfer techniques.

How well did you know this?
1
Not at all
2
3
4
5
Perfectly
2
Q

Module Overview

Family businesses may use estate freeze rules to assure retention of the business in the family at a minimum cost. The freeze rules allow the original owners, the older generation, to retain much of the present value and control, and have some source of revenue while the growth is shifted to the younger generation-the new managers of the business. It is used when the owners want to perpetuate the business or investment within the family.

Valuation techniques and discounts used for business interests reduce the value of the business or shares gifted to family members.

A business continuation planning process helps owners to make decisions regarding the continuation of the business after they end their involvement with the business. Business owners use the plan to decide who will take over the business, how their interest will be sold, how much the interest will be worth, and how the funds needed to purchase the interest being sold will be acquired.

A

Intra-family business transfer techniques are available to transfer business interests to family members in a most advantageous way.

To ensure that you have an understanding of business planning, the following lessons will be covered in this module:
* Estate Freeze Rules
* Valuation of Closely Held Businesses
* Business Continuation
* Intra-family Business Transfer Techniques

How well did you know this?
1
Not at all
2
3
4
5
Perfectly
3
Q

Section 1 - Estate Freeze Rules

How can Estate freeze techniques be performed?

A

Estate freeze techniques can be performed through installment sales of property, private annuities, premortem stock redemptions, preferred stock recapitalizations, a variety of gift planning techniques, and buy-sell agreements. The freeze is an important planning tool to assure retention of the enterprise in the family at minimum tax cost. Besides family businesses, estate freeze rules may also be used for family investments, particularly real estate.

There are two family business entities that estate freeze rules typically involve. The following table describes how these business entities would apply an estate freeze:

How well did you know this?
1
Not at all
2
3
4
5
Perfectly
4
Q

How would a Family Corporation apply an estate freeze?

A

Family Corporation: A closely held company where most of the voting shares are held by family members

Freeze Rules
* Senior generation typically retains control and an income stream by using voting preferred stock with a fixed liquidation value, usually par value.
* One or more classes of common stock are created either by:
* forming a new corporation, or
* recapitalizing an existing one.

The recapitalized common stock may be nonvoting or at least have limited voting rights.
All or part of this common stock is sold or gifted to the next generation.

How well did you know this?
1
Not at all
2
3
4
5
Perfectly
5
Q

How would a Family Partnership (LLC) apply an estate freeze?

A

Family Partnership (or LLC): A partnership where the controlling interest is held by family members

Freeze Rules
The senior family members can retain control and an income stream by:
* forming a new partnership, or
* restructuring an existing partnership to retain partnership interests that control the management of the business or investments. They can elect to receive preferred profit distributions, but the interest must have a fixed liquidation value.

The remaining non-control partnership interests may be sold or gifted to the next generation.

How well did you know this?
1
Not at all
2
3
4
5
Perfectly
6
Q

What is important about the retained stock or partnership interest of the senior generation?

A

The retained stock or partnership interest of the senior generation must have a fixed liquidation value.
* This makes it impossible for the value of those retained interests to grow (they are frozen).
* Though the retention of preferred dividends or profit distributions is not essential, it adds to the value of the retained interest.
* This increase in retained value reduces the value of the interests transferred (gifted) to the next generation, which results in a lower gift tax value.
* Therefore, the value of the retained interests becomes very important.

To ensure that you have an understanding of the estate freeze rule, the following topics will be covered in this lesson:
* IRC Chapter 14
* IRC Section 2701

Upon completion of this lesson, you should be able to:
* Define IRC Chapter 14 rules, and
* Describe IRC Section 2701 valuation rules.

How well did you know this?
1
Not at all
2
3
4
5
Perfectly
7
Q

Describe IRC Chapter 14

A

Chapter 14 of the Internal Revenue Code became effective on October 8, 1990. This Code section pertains to how to value interests in corporations and partnerships that are controlled by family members, generally where the family holds more than 50% of the value of the corporate stock or partnership interest.

Chapter 14 affects the gift tax valuation for lifetime transfers made to family members when partial interests are transferred.
* Therefore, the owner or family members retain some ownership of the property after the gift is made.
* Applicable family members are the transferor’s spouse, and the transferor and spouse’s lineal descendants, and their spouses.

Chapter 14 does not apply to:
* Transfers to third parties who are not related family members,
* A transfer of the entire business or property interest that the owner or applicable family members make to other family members, or
* Partial interest transfers made at death to family members.

How well did you know this?
1
Not at all
2
3
4
5
Perfectly
8
Q

There are 4 sections in IRC Chapter 14. Describe sections 2701 and 2702.

A
  • IRC Section 2701. Sec. 2701 applies to valuation of corporate and partnership interests designed for estate freeze purposes. In instances where there are capital interests that have a preference as to income distributions, the valuation of transfers, including recapitalizations among family members, may produce a gift tax.
  • IRC Section 2702. Sec. 2702 applies to interests in trusts. For example, Grantor Retained Income Trusts (GRITs), Grantor Retained Annuity Trust (GRATs), and Grantor Retained Unitrust (GRUTs) are split-interest trusts, and do not require consideration in valuing a closely held business.
  • IRC Section 2703. Sec. 2703 applies to any option, agreement, or other right to acquire or use the property at a price less than the fair market value of the property, without regard to such option, agreement or right, or any restriction on the right to sell or use such property.
  • IRC Section 2704. Sec. 2704 applies to lapsed voting and liquidation rights in a corporation or partnership. It applies if the individual holding such right immediately before the lapse, and members of such an individual’s family, hold control of the entity, both before and after the lapse. If lapsed voting or liquidation rights exist, then the amount of the transfer is determined by the value of the interest before the lapse minus the value after the lapse.
How well did you know this?
1
Not at all
2
3
4
5
Perfectly
9
Q

Practitioner Advice:

Practitioner Advice: Chapter 14 applies in situations where the older generation is giving something to the younger generation. This was Congress’ response to the abuse it felt was inherent in the corporate freeze arena. Under old laws, there were estate tax issues associated with the freezes where tax payments were deferred until death. Under Chapter 14, the gift tax rules hit inappropriately created freeze techniques with a current gift tax liability.

A
How well did you know this?
1
Not at all
2
3
4
5
Perfectly
10
Q

Describe IRC Section 2701

A

Once the senior family members of the business determine what they want to keep as a controlling interest of the company, they will give away the remaining interest to the younger generations of the family.
* How will this transfer of interest be treated?
* The Internal Revenue Code Section 2701 is titled, “Special valuation rules in case of transfers of certain interests in corporations or partnerships.”
* You must determine the value of any retained interest using the rules in IRC Sec. 2701 in order to determine the value of the remaining interest, and whether or not the transfer of interest is a gift for tax purposes.
* Under this section of the Internal Revenue Code, there is an attempt at determining whether the older generation is retaining any interest of value or giving it all away – if given all away, with only very tenuous retention of interest, the transfer tax kicks in form of a gift tax.

  • IRC Section 2701 also deals with the transfer tax treatment of the interest retained by the senior generation and how that retained interest will be valued for subsequent transfers as gifts or as inheritance.

IRC Section 2701 covered transfers include:
* Contributions in capital,
* Redemptions,
* Other changes in capital structure, and
* Transfers that increase the value of the property or applicable retained interest for the transferor and applicable family members.

Calculation of Residual Interest Value:
Total Value of Corporation or Partnership – Retained Interest = Value of Residual Interest Transferred to Family Member

How well did you know this?
1
Not at all
2
3
4
5
Perfectly
11
Q

What is the formula for Calculation of Residual Interest Value?

A

Calculation of Residual Interest Value:
Total Value of Corporation or Partnership – Retained Interest = Value of Residual Interest Transferred to Family Member

How well did you know this?
1
Not at all
2
3
4
5
Perfectly
12
Q

Define the Terms Used in IRC Section 2701

A

Applicable family members: For purposes of Section 2701, “family members” are the transferor’s spouse, lineal descendant of the transferor or spouse, and the spouse of any descendant.

Applicable retained interest: According to Section 2701(b), what is valued under the statute is an “applicable retained interest,” which is:
* A “distribution right,” if immediately after the transfer the transferor and “applicable family members” “control” the entity, or
* Liquidation, put, call, or conversion rights.

Distribution right: A distribution right is defined as a right to distributions from a corporation with respect to its stock or a partnership with respect to a partnership interest, except:
* Rights in connection with “junior equity interests,” defined as common stock or partnership interests under which rights to income and capital are junior to all other equity interests.
* “Liquidation, put, call, or conversion rights,” defined as any such right, or similar right, which affects the value of the transferred interest. However, the term does not include any right, which must be exercised at a specific time at a specific amount.
* Rights to guaranteed payments from a partnership defined in IRC section 707(c), which are certain payments determined without regard to partnership income.

Control means at least a 50 percent interest in a corporation by vote or value of the corporate stock, or at least 50 percent of the capital or profits in a partnership, or the holding of any partnership interest as a general partner. Control includes interests held by “applicable family members.” It also includes attribution through entities and interests held by brothers or sisters or lineal descendants of an individual.

Extraordinary payment rights are retained put, call, conversion rights, and rights to compel liquidation, or similar rights. A right falls within this definition only if it affects the value of the transferred interest.

How well did you know this?
1
Not at all
2
3
4
5
Perfectly
13
Q

Describe Applicable Retained Interest

A

Section 2701 is designed to determine whether a transfer of an interest in a corporation or partnership to a member of the transferor’s family is a gift based upon the applicable retained interest held by the transferor or an applicable family member immediately after the transfer.
* This is a transfer subject to gift tax rules.
* Sec 2701 is trying to determine whether so much has been given away that the transferor or applicable family member has retained no applicable retained interest in the enterprise.

Applicable retained interests include:
* Extraordinary payment rights: That is, a discretionary liquidation, put, call, conversion and rights to compel liquidation.
* Distribution rights: Right to distributions from a corporation’s stock or a partnership’s interest, and
* Qualified payment rights: Fixed-rate cumulative payment (that is, cumulative preferred stock periodic dividend) or payment which the transferor elects to treat as a payment. Qualified payments are fixed in time and amount.

The valuation of applicable retained interest is zero, except to the extent it consists of the right to receive qualified payments.
* For example, without a “qualified interest” the gift tax on the property transferred is based on the entire value of the business, even though the owner is only gifting a partial interest.
* The owner is considered to have a retained interest of zero in the company, therefore the closely held stock or partnership interest gifted is valued at the full value of the business.

The right to a qualified payment means the transferred interest is valued at fair market value.
* Therefore, if the business owner retains a “qualified payment” in the transaction, the partial interest gifted to family members is valued at its fair market value.

The reasoning behind retained interests is that the business owner has an interest with economic value that will be subject to a gift tax on future gifts, or subject to estate tax if held until death. Because the owner has this retained interest of value, the stock or partnership interests transferred to family members can be valued at their fair market value for gift tax purposes.
* A business owner who retains interests that have no economic value, a retained interest of zero, cannot be taxed on those assets in the future.
* Therefore, when a gift is made, the IRS will tax the transaction according to the full value of the business entity, not the value of the partial interest transferred.

How well did you know this?
1
Not at all
2
3
4
5
Perfectly
14
Q
A

For example, assume a closely held business owner has a large estate that includes a significant amount of stock from his business. The owner wants to transfer some stock to his children to reduce the value of his estate and to give his children ownership and the benefit of the stock’s future appreciation. He also wants to receive some income and retain complete control of his business.

With a Preferred Stock Recapitalization the owner might incorporate his business as a regular C Corp, with two classes of stock: voting preferred, which has a fixed par value, and non-voting common stock.

The owner keeps the voting preferred, with a cumulative right to receive a percentage of the company’s net profits annually.
* The cumulative preferred shares will satisfy the owner’s income needs while providing him with a “qualified payment” under Chapter 14: fixed-rate dividends paid periodically on cumulative preferred stock.
* The owner then gifts the non-voting common shares to his children. The children cannot control the management of the company, and this class of stock does not have a preference in the payment of income.
* The non-voting common shares are removed from the owner’s estate, which avoids estate taxation on the common stock’s value and any future appreciation.

Recapitalization also freezes the value of the preferred shares at par value, no matter how much the business grows, which further reduces the stock’s value in the owner’s gross estate.
* For instance, if the business owner’s company is worth $3 million and he gifts $1 million of common stock to his children and keeps $2 million of cumulative preferred shares, he will use up his applicable credit on the taxable gift and will pay income taxes on the fixed dividends he receives each year.
* When he dies, his company may be valued at $5 million, but only $2 million of the previously “frozen” cumulative preferred shares will be included in his gross estate.

How well did you know this?
1
Not at all
2
3
4
5
Perfectly
15
Q

Describe the Minimum Value Rule

A

If the transferor retains both a qualified payment right and an extraordinary right, the value of all rights is determined by assuming the extraordinary right is exercised in such a way to produce the lowest possible valuation.
* Since the value of applicable retained interest is zero except for qualified payments, then value of the interest gifted to the younger generation of the family business can be determined by:
* Value of family business - Value of retained qualified payment = Value of gift

It is possible that the value of the gift could be a small amount due to a large value for qualified payment. Regardless of the value of the gift determined by the subtraction, the minimum value that can be assigned to the transfer of junior equity interest is determined by:
* 10% of the total value of all equity interests in the partnership or corporation + the total indebtedness of the entity to the transferor or applicable family member

How well did you know this?
1
Not at all
2
3
4
5
Perfectly
16
Q

What is the minimum value that can be assigned to a transfer of junior equity interest if the total value of the equity interest is $10,000,000 and the total indebtedness is $5,000,000?
* $5,000,000
* $6,000,000
* $15,000,000
* $10,000,000

A

$6,000,000

  • The minimum value that can be assigned to the transfer of a junior equity interest is determined by:
    10% of the total value of all equity interests in the partnership or corporation + the total indebtedness of the entity to the transferor or applicable family member
  • (0.10 x $10,000,000) + $5,000,000 = $6,000,000
How well did you know this?
1
Not at all
2
3
4
5
Perfectly
17
Q

Describe Transfer Taxation of Distributions

A

If corporations or partnerships fail to make the required cumulative distributions or have noncumulative distributions that the transferor elected to treat as cumulative, then when the taxable event occurs, the taxable gifts or estate of the transferor are increased by the value of such unpaid distributions.

A taxable event is the death of the transferor if the applicable retained interest is included in his or her estate, or the transfer of such applicable retained interest. In addition, the transferor can choose to treat late payments as a taxable event.

This value of the distributions is determined on the assumption that all payments are made when due. Then the distributions are compounded as if reinvested at the same discount rate used in valuing the retained interest, less actual distributions paid, based on the date actually paid, but with a four-year grace period for late payment.

However, the additional amount subject to federal transfer tax is limited to the actual growth in the enterprise that was shifted.

If the transfer to a spouse is not taxable due to marital deduction, or because the transfer was for full and adequate consideration, then the distribution is not taxable. But the spouse will inherit the tax consequences on a subsequent taxable event.

Applicable family members other than the transferor are subject to the same taxable event rule with respect to their retained interests. If the applicable retained interest is transferred to such an applicable family member, he or she is subject to the same rules as to distributions accumulating after the transfer.

Qualified payments can be made in the form of debt instruments if they are for not more than four years and the compound interest is at a rate no less than the appropriate discount rate payable from the due date of the payment.

The value of the unpaid distribution is added to the taxable estate or taxable gifts of the transferor along with the value of the retained corporate stock or partnership interest.
* According to the IRC Section 2701, the amount subject to transfer tax is reduced by any amount (unpaid distributions) subject to transfer tax with respect to the same rights to prevent double taxation.

How well did you know this?
1
Not at all
2
3
4
5
Perfectly
18
Q

Describe Value of Retained Frozen Interests for Subsequent Transfers

A

Sec. 2701 only deals with the value of the retained interest for purposes of the valuation of a transferred interest at the time the interest is transferred.

The retained preferred stock or partnership interests are valued under normal rules for purposes of any subsequent transfers.
* This means any retained put, call, conversion, or liquidation right valued at zero for the purpose of determining the value of the gift under Sec. 2701 is subsequently valued when the retained interest is transferred.

This results in double taxation since an interest valued at zero when retained is subsequently valued at fair market value when the transferor dies or transfers the retained interest.
* To avoid this, there is an adjustment to the decedent’s and possibly the transferor’s spouse’s adjusted taxable gifts.
* This is done by reducing adjusted taxable gifts to reflect the amount by which the decedent’s taxable gifts were increased due to Sec. 2701 over the increase in the estate or adjusted taxable gifts attributable to the inclusion of the applicable retained interest in the estate.

How well did you know this?
1
Not at all
2
3
4
5
Perfectly
19
Q

Describe when a Freeze is a Viable Alternative?

A

Although a corporate or partnership freeze through the use of common and preferred stock or partnership interests is possible, many believe it has less utility in present times.
* In order to avoid a zero valuation for retained interests and minimize the value of the gift, it is necessary to make distributions with respect to the retained interests.

However, if the company is a C corporation, there is a double tax on corporate distributions (once at the corporate level and once at the shareholder level). The income tax will be paid now, while the estate tax is deferred, often until the death of a surviving spouse. If the entity is a partnership with a good cash flow, the freeze may be a good option because the distribution is only taxed once.

If the qualified payments are not made, the resulting inclusion of accumulated distributions in the transferor’s taxable gifts or estate could easily exceed the value of the retained interest for federal gift tax purposes.

How well did you know this?
1
Not at all
2
3
4
5
Perfectly
20
Q

Retained Interest and Taxable Gift Calculation Example:

Assume the applicable discount rate is 10%, and the retained dividend is 8%. Based on the assumption that the present value of the annual dividend is its fair market value, the value of the retained interest is 80% of the value of the transferred property. Thus, if the business is valued at $1,000,000, the retained interest is valued at $800,000, the gift at $200,000.

If the dividends are not paid for 10 years, and the grace period does not apply, the value of these unpaid dividends plus accrued interest at the same 10% is approximately $1,275,000 (n = 10, i = 10, PV = 0, PMT = 80,000, solve for FV). This would be the amount of the taxable gift under Sec. 2701(d)(1).

A

In the example, the amount included in the taxable transfers of the transferor cannot exceed the actual growth in the business.

The Congressional Committee Reports acknowledge that all existing discounts are preserved. This makes it possible to state that the value of the transferred interest for federal gift tax purposes must be reduced to reflect discounts for minority interests and lack of marketability.

How well did you know this?
1
Not at all
2
3
4
5
Perfectly
21
Q

How do you Calculate Taxable Gift Value Reflecting Discount?

A
  • Determine the FMV of all family-held interests in the entity.
  • Subtract all senior equity interests held by the family other than applicable retained interests held by the transferor or applicable family members, with a pro-rata adjustment for any control premium, followed by the subtraction of the value of all applicable retained interests held by the transferor and applicable family members.
  • Allocate the remaining value among the transferred interests and any other family-held junior or subordinate interests.
  • Reduce value for consideration received, and for any minority or similar discounts (including a discount for lack of marketability) determined by subtracting the FMV of the family-held interests from the value of the transferred interest determined without regard to Sec. 2701.
How well did you know this?
1
Not at all
2
3
4
5
Perfectly
22
Q

What are Alternatives to the Freeze?

A

If the transferor retains common stock or partnership interests and transfers the preferred interests, Sec. 2701 does not apply.
* This is sometimes called a reverse freeze.
* When the transferor dies, the value of the common stock will reflect growth.
* However, the common stock would be substantially discounted in value because of the burden of the preferred stock with its prior claim to dividends and liquidation proceeds.

A variety of other techniques may be used to freeze growth. These include:
* Installment sales of business interests to family members
* Use of the private annuity, and
* Premortem stock redemptions or partnership liquidations of interests of senior family members, giving senior family members a cash flow and transferring the growth in the value of the enterprise to the next generation.

Under some circumstances, if the transferor receives a debt instrument issued by a family partnership or corporation, the IRS may seek to treat it as retained preferred stock.

How well did you know this?
1
Not at all
2
3
4
5
Perfectly
23
Q

Section 1 - Estate Freeze Rules Summary

Freeze techniques are used in closely held family corporations and partnerships to retain control and authority. The older generation may pass on only the additional benefits to their successors, the younger generation. Or they may create common stock that has nonvoting or limited voting rights. The retained stock or partnership interests of the senior generation have to have a limited liquidation value. Valuation of businesses and properties transferred is a blend of the estate planning, appraiser’s skills, IRC, and court judgments.

In this lesson, we have covered the following:
* Chapter 14: IRC Chapter 14 is used to value interests in corporations and partnerships. Chapter 14 affects the gift tax value of partial interests of corporate stock or partnership interests that are transferred to applicable family members, who hold more than 50% of the value of the corporate stock or partnership interest.

A
  • Section 2701 determines whether interest transfers in partnerships or corporations to family members are gifts. It applies to the applicable retained interest, which consists of: the extraordinary payment right, the distribution right, and the qualified payment right. The minimum value of a gift is ten percent of the total value of all equity interests, plus the total indebtedness of the entity to the transferor. If a taxable event occurs, the taxable gifts or estate of the transferor increases by the value of unpaid distributions. Corporate distributions are liable for double tax. As preferred stock is involved, the use of freeze eliminates qualification for the S-Corp election. The various freeze techniques include: installment sales of business interests to family members, use of a private annuity, and pre-mortem stock redemptions or partnership liquidations of interests.
How well did you know this?
1
Not at all
2
3
4
5
Perfectly
24
Q

Harold Bates would like to gift some part of his stock in the corporation to an applicable family member, after which he would retain some interest in the corporation. To which of the following members could he gift a part of his stock? (Select all that apply)
* Harold’s wife Sandra
* Harold’s great-aunt Hilda
* Harold’s son Jake
* Harold’s brother Michael
* Harold’s niece Cindy

A

Harold’s wife Sandra
Harold’s son Jake

  • Harold can gift the stock to Sandra and Jake. Under Sec. 2701, family members are the transferor’s spouse, lineal descendant of the transferor or spouse, and the spouse of any descendant
How well did you know this?
1
Not at all
2
3
4
5
Perfectly
25
Q

Lynn is an owner of a closely held business that is appreciating rapidly. She decides to transfer $400,000 of stock to her children who work in the family business to reduce the value of her estate, and have her children benefit from the company’s growth. Lynn wants to receive income from the business but she does not structure the income payments as qualified payments. Therefore, her retained business interest is valued at zero.
* False
* True

A

True
* Under Chapter 14, a qualified payment is needed to have the stock transferred to her children valued at $400,000. With a retained business interest of zero, the gifted shares are valued at the full value of Lynn’s business interest. If Lynn had recapitalized her stock into non-voting common and cumulative voting preferred shares, and gifted the common stock to her children, she would have “qualified payments” and the common stock would be valued at $400,000 when transferred.

How well did you know this?
1
Not at all
2
3
4
5
Perfectly
26
Q

Julia Douglas at 55 would like to hand over the reigns of her multi-billion dollar enterprise to her son Eddie. But as he is very new to the business, she would like to give him some more time to learn the tricks of the trade. She plans to freeze growth of the enterprise and pass on the benefits to Eddie. Which of the following methods can she adopt to bring about the freeze? (Select all that apply)
* Installment sales of business interests to family members
* Use of the private annuity
* Preferred stock recapitalization
* Buy-sell agreements
* The transferor retains common stock or partnership interests and transfers the preferred interests

A

Installment sales of business interests to family members
Use of the private annuity
Preferred stock recapitalization
Buy-sell agreements
* There are a variety of freeze growth techniques that Julia can use. Installment sales, private annuities and preferred stock recapitalizations give business owners cash flow and they transfer the growth in the value of the enterprise to the next generation. Buy-sell agreements also freeze the value of the business interest for estate tax purposes.

How well did you know this?
1
Not at all
2
3
4
5
Perfectly
27
Q

Match the section with the correction description:
Sec. 2701
Sec. 2702
Sec. 2703
Sec. 2704
* Applicable to lapsed voting and liquidation rights in a corporation or partnership.
* Applicable to valuation of corporate and partnership interests designed for estate freeze purposes
* Applicable to any agreement or right to acquire or use the property at a price less than the fair market value
* Applicable to interests in trusts

A
  • Sec. 2701 - Applicable to valuation of corporate and partnership interests designed for estate freeze purposes
  • Sec. 2702 - Applicable to interests in trusts
  • Sec. 2703 - Applicable to any agreement or right to acquire or use the property at a price less than the fair market value
  • Sec. 2704 - Applicable to lapsed voting and liquidation rights in a corporation or partnership.
How well did you know this?
1
Not at all
2
3
4
5
Perfectly
28
Q

Section 2 - Valuation of Closely Held Business

To be effective, what 3 elements must an estate freeze plan contain?

A

Lower valuation for the business interest may result from interest that does not fully participate in the future growth of the business, such as preferred stock and frozen partnership interests, and long-term installment sales. A failure to properly undertake valuation planning for a family business can leave an estate at the mercy of the IRS and may make it impossible to retain the enterprise.

To be really effective, any estate freeze plan must contain the following elements:
* A present transfer of interests in the business from family members in the older generation to those in younger generations, either by sale, gift, or both.
* A shift of all or part of the future growth of the enterprise to the younger generation.
* The owners of the family business should consider making full use of their applicable credit and even their GST tax exemption for lifetime transfers of family business interests.

On the other hand, retention of the value of business interests in the estates of the older generation may facilitate stock redemptions, particularly under IRC Sec. 303, deferred payment of tax under IRC Sec. 6166, and special-use valuation for real property under IRC Sec. 2032A, to meet the minimum percentage requirements in the gross and adjusted gross estates.

How well did you know this?
1
Not at all
2
3
4
5
Perfectly
29
Q

Describe Valuation Discounts

A

The IRS has recognized that valuation of interests in closely-held businesses and family limited partnerships must take into account the difficulty of transferring property that has a limited market, or property that has limited rights.
* Discounts can be applied when these business interests are transferred to family members to reflect the proper valuation of these limited interests.

The two most common discounts available for family and closely-held businesses are the discounts for
* Lack of marketability, and/or
* A minority interest.

A lack of marketability discount is used when valuing transfers of business interests to junior family members during the business owner’s life or at death. A discount is allowed because investors are not interested in closely held stock or family limited partnership shares, and the cost of taking this stock public or selling it on an exchange to potential investors is very expensive. Therefore, a discount for the lack of marketability is allowed by the IRS.

In valuing interests in corporations, which hold investment assets such as real property, marketable securities, and coins, consider whether there are potential capital gains that will be taxed at the corporate level.
* The existence of potential capital gains is considered a factor that increases the lack of marketability discount.

A minority interest discount is allowed when transferring business interests to minority shareholders because these shareholders have no influence or control over business operations or management policy. Specifically, minority shareholders cannot compel dividend payments or obtain the company’s assets through liquidations, mergers, or sales.
* A discount is applied to a percentage of the appraised value of the stock which reduces its value for a gift or estate tax purposes.

A lack of marketability discount and a minority interest discount can be combined together when calculating a discount on shares of closely held stock.
* Typically the IRS will scrutinize discounts that exceed the 35-45% range, so business valuations should be conducted by professional appraisers.

How well did you know this?
1
Not at all
2
3
4
5
Perfectly
30
Q

Describe Lock-In Discount

A

A discount that reflects a circumstance in which a limited partner cannot withdraw from the partnership and is locked into his investment, is known as a lock-in discount.

For partnerships, always consider whether the partnership interest can be liquidated through the enforcement of withdrawal rights.
* In some states, if the partnership agreement does not provide a time for when capital will be returned, a limited partner is given the right to withdraw from the partnership six months after giving notice to the general partner.

In other states, applicable state law prohibits the return of capital to a limited partner except upon the occurrence of certain events specified in the partnership agreement, for example, upon expiration of the partnership term.

If the partnership agreement or state law does not confer upon the limited partner any right to withdraw capital, then the interest may remain in the partnership until the expiration of the partnership term, often 35-50 years in length.
* In such a case, the limited partner is locked in to maintain his investment in the partnership and a lock-in discount is appropriate.

How well did you know this?
1
Not at all
2
3
4
5
Perfectly
31
Q

Describe Key Person Discount
and Blockage Discount

A

A key-person discount is allowed to reduce the value of an estate to less than fair market value upon the death of a key employee in a closely-held business. The key person in many businesses is the founder of the business, who typically owns most of the stock and controls the management of the company.
* Since the value of the company’s stock would sharply decline at the owner’s death, the IRS may allow a discount for the impact that this loss might have on the business.

A blockage discount is allowed by the IRS if the immediate sale of a substantial amount of publicly traded stock that the decedent owned would adversely affect the stock’s market price.
* The blockage discount would reduce the value of the stock held in the decedent’s estate, and reduce the estate tax liability.

How well did you know this?
1
Not at all
2
3
4
5
Perfectly
32
Q

Match the following with it’s correct descrption:
Minority Interest
Lack of Marketability
Key Person Discount
Blockage Discount
* Sale of sizable amount of publicly traded stock at the owner’s death which depresses its market price.
* Closely held business stocks with no readily available market.
* Discount for the loss to the business due to the owner’s death.
* A shareholder has less than majority interest and has no control over the business.

A
  • Minority Interest - A shareholder has less than majority interest and has no control over the business.
  • Lack of Marketability - Closely held business stocks with no readily available market.
  • Key Person Discount - Discount for the loss to the business due to the owner’s death.
  • Blockage Discount - Sale of sizable amount of publicly traded stock at the owner’s death which depresses its market price.
How well did you know this?
1
Not at all
2
3
4
5
Perfectly
33
Q

Describe the Fractional Interest Discount

A

A fractional interest discount or a “co-ownership discount” is available in the decedent’s estate for real property that is owned with another party, who is unwilling to sell their partial interest to a third party or to the decedent’s estate, or who will not buy out the decedent’s partial interest in the property.
* The co-owner of the property cannot be the decedent’s spouse, heirs, business partners, or joint tenants under JTWROS.

The courts recognize that the value of a fractional interest in real property may be less than the pro-rata value of the same real property if it were held as full fee simple property.
* The discount is a percentage of the value of the real property included in the decedent’s estate, which reduces the decedent’s gross estate.

Among the factors cited by the courts in justifying the application of a discount to the pro-rata value of a fractional interest in real property are the following:
* The owner of a fractional interest has greater difficulty in finding a ready market for the sale of the interest due to the fact that a buyer of the interest will have to share ownership with another.
* The owner of a fractional interest cannot sell the fee interest in the property or lease the property without the consent of the holder of the remaining interests.
* The owner of a fractional interest by himself generally cannot obtain a loan from normal sources of credit secured by only a partial interest in the real property.
* The owner of a fractional interest may not have the right to exclusive use of the property so as to put it to its highest and best use.
* An action by an owner of a fractional interest to partition real property does not guarantee that the partitioning co-owner will receive property whose utility is equal to that of the entire parcel from which the partition took place.
* The cost of bringing an action to partition real property is often substantial and fee interest in a portion of the property received may be worth less than the proportionate share of the whole property or the sales proceeds. If the property is not fairly partitioned it may not equal the property’s fair market value.
* The existence of an undivided fractional interest is a legal matter affecting title to real property and often cannot be solved without obtaining legal counsel and commencing legal proceedings at significant cost.

For these reasons, many courts have allowed a discount to the proportionate value of real property in order to arrive at the value a willing purchaser would be willing to pay for an undivided fractional interest in real property. If the taxpayer asserts that a discount from the proportionate value of the entire property is warranted, he or she must prove that a willing buyer would purchase the property and a willing seller would sell the property at a price that is less than its proportionate value.

The extent of ownership rights is determined under state law. Though the IRS Valuation Manual states that the fractional interest discount is generally based on the cost of dividing the land, such as survey costs, court costs, and legal fees, the following other factors must be considered:
* Size of the fractional interest
* Number of owners
* Size of tract and likelihood of partition
* Use of land
* Access to financing

How well did you know this?
1
Not at all
2
3
4
5
Perfectly
34
Q

What are the pitfalls to avoid in Fractional Interest Discount?

A

In the valuation of closely-held businesses, there are some potential pitfalls.
* For example, in planning for discounted asset values, the calculation of adjusted taxable gifts may force the value of all lifetime gifts to be reconsidered. The result of the reconsideration could increase the estate’s tax bracket.

  • Another pitfall may come in over-reliance on expert valuations. The following cases illustrate how the courts are closely examining the approach used by experts in valuing business interests and the qualifications of the appraisers:
  • In Estate of Murphy, TC Memo 1990-472, 18 days before death the decedent transferred less than 1% to each of two children to bring her percentage below 50%, and her estate claimed a minority discount. The Tax Court allowed a discount for lack of marketability, but rejected the minority discount, noting that the decedent gave up control on paper, but not in reality.
  • In Estate of Campbell, TC Memo 1991-615, the court emphasized the importance of evaluating both earnings and net asset values in arriving at the actual value and allowed a substantial discount.
  • In Estate of Berg, TC Memo 1991-279, a 60% discount in the value of the closely-held stock was claimed to represent both minority and lack of marketability discounts. The appraisal was by petitioner’s CPA, Mr. Whalen, an experienced practitioner who had served on the faculty of several universities and had testified as an expert witness in several cases on lack of marketability and minority discounts, but as the court noted, with no formal education as an appraiser.
How well did you know this?
1
Not at all
2
3
4
5
Perfectly
35
Q

Describe Controlling Interest

A

If the business interest being valued is a controlling interest, then you must add a premium to the valuation to reflect control. The IRS has applied this concept to situations where the transfers to individual family members were of minority interests, but they could collectively exercise control over the business entity.

Controlling Interest Example:
An owner of a closely held business owned 100% of the stock in a family corporation. He transferred 30% to each of the three children and 5% to his spouse. The gifts were valued at net asset value less a 25% discount for minority interest and lack of marketability.
* Since the blocks were transferred simultaneously, each of the children had a swing vote characteristic.
* This meant that the owner of any 30% block could join with the owner of any other 30% block to exercise control.
* As the controlling interest increases, premiums should be added to each block accordingly.

How well did you know this?
1
Not at all
2
3
4
5
Perfectly
36
Q

Section 2 - Valuation of Closely Held Business Summary

In this lesson, we have covered the following:
* Valuation of a Closely Held Business cross-purchase or redemption plans and partnership liquidation agreements have a substantial impact on the valuation of a business, including restrictive agreements. Lower valuation for the business interest may result from retained interests that do not fully participate in the future growth of the business, such as preferred stock and frozen partnership interests, and long-term installment sales. Valuation discounts are available for transfers made to junior family members, such as minority discounts and lack of marketability discounts, which reduce the value of the gifted shares. Other discounts are available to reduce a person’s gross estate such as key person discounts, blockage discounts, and fractional interest discounts.
* Valuation Discounts are available for transfers of closely held business interests and family limited partnership interests to family members. Discounts can be taken for lack of marketability and minority interests that reflect the limited public value of the gifted shares, and the limited partner’s lack of control and influence over the company.

A
  • Lock-In Discount is permitted when a limited partner cannot withdraw from a partnership and is therefore “locked-in” to maintain his investment.
  • Other Discounts can be taken to reduce the value of a decedent’s estate after 2010. A key-person discount is taken upon the death of a key employee in a closely held business to compensate for the loss to the business. A blockage discount is permitted if a sizeable amount of a decedent’s publicly traded stock would depress the stock’s market price if sold.
  • Fractional Interest Discount or “co-ownership” discount is available at the owner’s death when the fractional interest owned in the property cannot be readily sold.
  • Avoiding Pitfalls such as over-reliance on expert valuations, and having gifted property brought back into the estate tax calculation as adjusted taxable gifts.
  • Controlling Interests: a premium is added to each unit of stock transferred to limited partners when the collective value of their shares control the business or partnership.
How well did you know this?
1
Not at all
2
3
4
5
Perfectly
37
Q

Choose the most appropriate valuation technique to reduce federal estate taxes.
* Fractional interest discount can be taken for property the decedent held as tenants in common with a neighbor, who refuses to sell the property.
* A blockage discount is available when an executor sells 100% of the decedent’s partnership shares to the remaining partner.
* A minority interest discount is available for the decedent’s ownership in a house he co-owns with his father.
* A key man discount can be used in the limited partner’s estate.

A

Fractional interest discount can be taken for property the decedent held as tenants in common with a neighbor, who refuses to sell the property.

  • A fractional interest discount can be taken at the property owner’s death when the neighbor refuses to sell his partial interest in the tenancy in common property.
  • A blockage discount only applies to sales of publicly traded stock that would depress market price if sold.
  • A minority interest in a home is not a closely held business interest.
  • A key man’s discount is only available for the loss to the business upon a key employee’s death.
How well did you know this?
1
Not at all
2
3
4
5
Perfectly
38
Q

Which of the following statements are correct? (Select all that apply)
* A lack of marketability discount can be combined with a minority interest discount when valuing limited partnership interests.
* A minority discount is available because an interest in a closely held business is more difficult to sell than publicly traded securities.
* A lack of marketability discount is available for all interests, whether it is a minority or a majority interest.
* A minority discount and lack of marketability discount can be applied to publicly traded stock sold on a major exchange.

A

A lack of marketability discount can be combined with a minority interest discount when valuing limited partnership interests.
A lack of marketability discount is available for all interests, whether it is a minority or a majority interest.
* A lack of marketability discount can be combined with a minority interest discount for valuing limited partnership shares.
* A lack of marketability discount is available for all interests, whether it is a minority or a majority interest.
* A minority discount is available since minority ownership cannot influence the business, compel dividends, or liquidate the company.
* Both discounts apply to shares in closely held businesses, not to publicly traded shares on a stock exchange.

39
Q

Section 3 - Business Continuation

Death, disability, or retirement of a key executive can create a succession crisis. If the owner of a business died, would his family be able to operate the business successfully? If his successors had the opportunity to buy the business, where would they get the money? If a partner were no longer there, could the surviving successor afford to buy his or her share?

One of the biggest apprehensions of a business owner is the fate of his or her business after he or she passes away or can no longer be part of the business.

With a carefully constructed business continuation plan, the business owner can create a solid foundation for the future of his or her business and the people who depend on it.

A

To ensure that you have an understanding of business continuation, the following topics will be covered in this lesson:
* Defining Business Continuation
* Stock-Redemption Agreement, Tax Implications
* Section 303 Stock Redemption

Upon completion of this lesson, you should be able to:
* Define the business continuation agreement,
* Describe the use and requirements of the business continuation plan,
* Distinguish between cross-purchase and stock redemption agreements,
* Explain sinking fund,
* Describe the uses of stock redemption,
* List IRC requirements for stock redemption, and
* Explain the tax implications of stock redemption.

40
Q

Define Business Continuation

A

A business continuation agreement is an arrangement for the disposition of a business interest in the event of the owner’s death, disability, retirement, or upon withdrawal from the business.

Business continuation agreements take a number of forms:
* An agreement between the business itself and the individual owners, either a corporate stock redemption agreement or partnership liquidation agreement, frequently called an entity plan,
* An agreement between the individual owners, a cross-purchase or criss-cross agreement,
* An agreement between the individual owners and a key person, family member, or outside individual such as a third-party business buyout agreement, or
* A combination of the foregoing.

For corporations, the most common types of business continuation agreements are stock redemption plans, also called stock retirement plans, and shareholder cross-purchase plans.
* The distinguishing feature of the redemption agreement is that the corporation itself agrees to purchase or redeem the stock of the withdrawing or deceased shareholder.
* In a cross-purchase plan, the individuals agree between or among themselves to purchase the interest of a withdrawing or deceased shareholder.

In the case of a partnership, the business continuation agreement is the partnership liquidation agreement, where the partnership in effect purchases the interest of the deceased or withdrawing shareholder by distributing assets in liquidation of the partner’s interest, or the partners agree to a cross-purchase similar to the corporate cross-purchase plan.

41
Q

Match the term with the correct description.
Stock Redemption Agreement
Cross-Purchase Agreement
Business Buyout Agreement
Partnership Liquidation Agreement
* Individuals agree among themselves to purchase the interest of a withdrawing or deceased shareholder.
* Identifies the business continuation plan for a partner’s interest in case of death or withdrawal.
* Corporation purchases or redeems stock of the withdrawing or deceased shareholder.
* Key person, family member, or outside individual to purchase interest.

A
  • Stock Redemption Agreement - Corporation purchases or redeems stock of the withdrawing or deceased shareholder.
  • Cross-Purchase Agreement - Individuals agree among themselves to purchase the interest of a withdrawing or deceased shareholder.
  • Business Buyout Agreement - Key person, family member, or outside individual to purchase interest.
  • Partnership Liquidation Agreement - Identifies the business continuation plan for a partner’s interest in case of death or withdrawal.
42
Q

When can you use a business continuation agreement?

A

You can use a business continuation agreement:
* When a guaranteed market must be created for the sale of a business interest in the event of death, disability, or retirement.
* When it is necessary or desirable to peg the value of the business for federal and state death tax purposes.
* When a shareholder or partner is unable or unwilling to continue running the business with the family of a deceased co-owner.
* When the business involves a high amount of financial risk for the family of a deceased owner and it is desirable to convert the business interest into cash at the owner’s death.
* When it is necessary or desirable to prevent all or part of the business from falling into the hands of outsiders. This could include a buyout of an owner’s interest in the event of a divorce, disability, or insolvency if there is a danger a business interest would be transferred to a former spouse or creditors.
* Where it is desirable to lend certainty to the disposition of a family closely-held business. Rather than relying on will provisions of a parent to transfer the business interest, a binding buy-sell agreement between parent and child could be used.

43
Q

What are the Requirements in a Business Continuation Agreement?

A

A written agreement is drawn stating the purchase price, terms, and funding arrangements. The agreement obligates the retiring or disabled owner or owner’s estate to sell the business either:
* To the business itself
* To the surviving owners
* To a third party nonowner
* To a combination of parties

Occasionally, the agreement combines the types of obligations. For example, the agreement may give other owners the option to purchase the stock or partnership interest. However, it provides that if the owners fail to exercise that option, the interest must be redeemed or liquidated by the corporation or partnership.

Conversely, the agreement may provide that if the entity cannot purchase the interest, the remaining owners have either an obligation or option to do so. Such agreements need to be carefully drafted to avoid a situation where the entity discharges an obligation of the other owners to purchase the interest that could have adverse tax consequences to the other owners.

For example, if the shareholders under the agreement are obligated to buy the stock of a deceased shareholder, and the corporation redeems the stock, the amount of the obligation of which the remaining shareholders are relieved could be considered a taxable dividend to them.

The buy-sell agreement specifies the event triggering the respective obligations. Generally, that event is death, disability, or retirement of the owner. However, it could include divorce or insolvency, and events such as loss of a professional license by an owner or conviction of an owner of a crime. Valuation may be based on several factors, like book value, asset value, formula value, or some agreed amount.

Funding pertains to how the promises under the agreement will be financed.
* Generally, in a redemption or liquidation agreement, the business purchases, owns, and is the beneficiary of life and disability income insurance on each person who owns an interest in the business.
* If a cross-purchase agreement is used, then each business associate purchases, owns, and is a beneficiary of each other’s life and disability insurance policies.

The buy-out price is also important in buy-sell arrangements and set prices, without formula adjustments, can bring in the application of Chapter 14 issues.

44
Q

Identify the event(s) that can trigger business continuity contingencies. (Select all that apply)
* Death
* Retirement
* Disability
* Divorce
* Loss of professional license to continue performing business-related tasks

A

Death
Retirement
Disability
Divorce
Loss of professional license to continue performing business-related tasks
* The buy-sell agreement specifies the triggering event. Generally, that event is death, disability, or retirement of the owner. It can include divorce or insolvency, and events such as loss of a professional license by an owner or conviction of an owner of a crime.

45
Q

Example of Stock Redemption Plan

Herb and his son Steve are co-owners of a business. As part of a business continuity planning strategy, they drafted a buy-sell agreement that states the business will purchase the owners’ shares in case of disability, retirement, or death. Assume Herb and Steve are equal stockholders in a business valued at $5,000,000. The business purchases $2,500,000 of life insurance on both men.

Let’s say Herb becomes disabled prior to retirement.
* Per the buy-sell agreement of his business, Herb will receive his full salary for one year.
* At the end of a year, Herb’s interest would be sold to the business.
* The business would pay at least $250,000, that is, 10 percent of $2,500,000, as a down payment to Herb.
* The business would also issue Herb a 10-year note for the remaining value of the stock.
* The note is secured by his stock in an escrow account.
* The business would pay interest on the note.
* The rate would be the safe harbor rate required to avoid the unstated interest rules.
* In order to help pay off the note, the business would apply for and name itself beneficiary of disability income insurance.

If Herb was never disabled and retired from the business when he turned 70 years old, he would sell his stock to the business.
* The business would pay Herb at least $250,000, that is, 10 percent of $2,500,000, as a down payment, plus give him a 10-year note, secured by his stock for the remaining amount.
* The business would pay interest on the note at the safe harbor rate needed to avoid the unstated interest rules.
* The cash value of the life insurance policy on Herb’s life could be used to help finance the down payment.
* One limiting factor could be that the corporation can redeem its stock only to the extent it has earned a surplus.

If Herb died prior to retiring:
* Herb’s stock passes to his estate.
* The life insurance proceeds on Herb’s life are paid to the business.
* Then the business pays the cash proceeds to Herb’s estate according to the agreement.
* In return for the cash, Herb’s executor transfers the stock to the business.
* Steve ends up with ownership of all the outstanding voting stock because the stock owned by the business itself is not entitled to vote.
* There should be no gain or loss recognized on this transaction to the decedent shareholder’s estate since the full value of the shares were included in the estate, they received a stepped-up basis and were then sold for their higher market value.

A
46
Q

Example of Cross-Purchase Agreement

In the case of a cross-purchase agreement, each business associate purchases, owns, and is the beneficiary of a life and disability income insurance policy on the other owners.
* At death, the policies the decedent owned on the other business associates will be included in the gross estate at replacement cost value, since the decedent was the owner but not the insured of the other policies.
* The value of the life insurance owned by the other business associates on the decedent’s life will not be included in the decedent owner’s estate.

To continue from our earlier example, Herb and Steve are equal stockholders in a business valued at $5,000,000. Herb purchases a $2,500,000 life insurance policy and a disability income policy on Steve’s life. Steve purchases policies in equal amounts on Herb’s life.

Assuming Herb dies first:
* Herb’s stock passes to his estate.
* The insurance proceeds on Herb’s life are paid directly to Steve.
* Steve then pays cash to Herb’s estate according to the cross-purchase agreement.
* In return for the cash, Herb’s executor transfers stock to Steve.
* Steve becomes the sole owner of the corporation.

In the event of Herb’s disability,
* he receives his full salary for one year assuming he is totally disabled.
* At that time, Herb sells his business interest to Steve.
* Steve pays at least $250,000, that is, 10 percent of $2,500,000, as a down payment to Herb.
* Steve also gives Herb a 10-year note, secured by his stock for the balance.
* Steve pays interest on the note at the safe harbor rate needed to avoid the unstated interest rules.
* During this time, Steve receives income from the disability income insurance policy purchased on Herb’s life. This can be used to pay off the note.

At Herb’s retirement,
* Herb will sell his stock to Steve.
* Steve will pay at least $250,000, that is, 10 percent of $2,500,000, as a down payment to Herb.
* Steve will also give Herb a 10-year note, secured by his stock for the balance.
* Steve will pay interest on the note at the safe harbor rate to avoid the unstated interest rules.
* Steve can use the cash value of the policy he owns on Herb’s life to help provide the down payment.

A

Practitioner Advice:
* The stock redemption arrangements are advantageous in the event of the principal owner’s disability and permit the use of corporate funds for the purchase and are usually more cost-effective from a tax standpoint than the use of personal funds.
* However, cross-purchase agreements give the other owners more control over a successor who can manage the business and can also provide big tax advantages for the new owners.
* It is extremely important that the value of the business interest for estate tax purposes is pegged by the agreement and usually the company’s tax advisor is consulted about the matter.
* Once the value is pegged, the business can’t be sold for more than the price set by the stock purchase agreement either before or after the death of the principal business owner.
* It is better to have an initial appraisal and then a formula appraisal mechanism in the agreement to determine the fair market value of the stock to avoid Chapter 14 issues.

47
Q

Practitioner Advice:

Practitioner Advice:
* The stock redemption arrangements are advantageous in the event of the principal owner’s disability and permit the use of corporate funds for the purchase and are usually more cost-effective from a tax standpoint than the use of personal funds.
* However, cross-purchase agreements give the other owners more control over a successor who can manage the business and can also provide big tax advantages for the new owners.
* It is extremely important that the value of the business interest for estate tax purposes is pegged by the agreement and usually the company’s tax advisor is consulted about the matter.
* Once the value is pegged, the business can’t be sold for more than the price set by the stock purchase agreement either before or after the death of the principal business owner.
* It is better to have an initial appraisal and then a formula appraisal mechanism in the agreement to determine the fair market value of the stock to avoid Chapter 14 issues.

A

Practitioner Advice:
* Many authorities suggest that a third party such as an irrevocable trust owns the life insurance.
* At the shareholder’s death, the proceeds are paid to the policy owner who can then make a fully secured loan to the corporation.
* The corporation could then redeem the stock under Sec. 303.

The three advantages of this technique are:
* The insurance proceeds don’t increase the value of the corporation for estate tax purposes,
* Cash values cannot trigger an accumulated earnings tax problem, and
* Neither cash values nor death proceeds can trigger an alternative minimum tax problem.

48
Q

Describe the Tax Implications of Stock Redemption Agreements

A

Assuming the corporation is the owner and beneficiary of the policies, the value of the insurance is not includable as insurance proceeds in his gross estate for federal estate tax purposes. However, the insurance proceeds will be considered in valuing the decedent’s interest in the business unless there is a valid agreement fixing the price and the proceeds are excluded from the purchase price under the terms of the agreement.

The buy-sell agreement generally establishes the value of the business for federal estate tax purposes if:
* The estate is obligated to sell at the decedent shareholder’s death, or the estate is obligated to offer the decedent’s shares at his death at the agreement price,
* The agreement prohibits the shareholder from disposing of his interest during his lifetime without first offering it to the corporation at no more than the contract price,
* The price was fair and adequate at the time the agreement was made and resulted from a bona fide arm’s length transaction between the parties, and
* The price was fixed by the terms of the agreement, or, preferably, the agreement contained a formula for determining the price.

49
Q

Describe Insurance Premiums and Sinking Fund

A

Life insurance and disability income premiums used to fund the purchase of an owner’s interest are not deductible to the corporation.
* On the other hand, the death proceeds and disability income proceeds will be received by the insurance corporation on an income tax-free basis.
* Also, premiums paid by the corporation on the insurance policies are not taxable to shareholders.

If in any tax year the corporation’s taxable income is retained either for paying insurance premiums or for any other purpose in excess of the amount of the $250,000 accumulated earnings credit ($150,000 in the case of certain personal service corporations), the corporation should show that such excess retentions are necessary to meet reasonable needs of the business. Such excess retentions that are beyond reasonable business needs may attract tax.

Where an uncommitted key individual life insurance policy of appropriate amount and type is used to shift the risk of the loss of a key person’s services, it generally does not cause or aggravate an accumulated earnings tax problem. Likewise, even though death proceeds do increase earnings and profits to the extent they exceed premiums paid, they should not, per se, cause or aggravate an accumulated earnings tax problem. Term, whole life, or a similar low cash value policy should be used. Furthermore, wherever possible the redemption should be effected in the same fiscal year that death occurs.

Besides insurance proceeds and 10-year notes (installment or self-canceling installment notes), a corporation can also establish a sinking fund and pay periodically to the fund with the purpose of someday using it to pay for stock redemption.
* A sinking fund can also be established to fund the repayment of the 10-year notes.

50
Q

Practitioner Advice:

Practitioner Advice:
* Many authorities suggest that a third party such as an irrevocable trust owns the life insurance.
* At the shareholder’s death, the proceeds are paid to the policy owner who can then make a fully secured loan to the corporation.
* The corporation could then redeem the stock under Sec. 303.

The three advantages of this technique are:
* The insurance proceeds don’t increase the value of the corporation for estate tax purposes,
* Cash values cannot trigger an accumulated earnings tax problem, and
* Neither cash values nor death proceeds can trigger an alternative minimum tax problem.

A
51
Q

Describe Dividend Distribution

A

The biggest potential problem in a corporate stock redemption agreement is the possibility that the redemption will be treated as a dividend distribution.
* The IRC states that no matter how the parties label a transaction, distribution of money or property in the redemption of stock by a corporation will generally be treated as a dividend subject to ordinary income taxation.
* If the distribution or redemption meets certain exceptions to the general rule, the seller can often avoid ordinary income treatment and can usually have the transaction treated as a capital gain.

The most common exception is the redemption of all of a shareholder’s stock.
* This complete termination of interest will allow a corporate stock redemption not to be treated as a dividend.
* For example, Hank and Sam, unrelated parties, own all of the stock in a closely held corporation. Hank dies and his stock passes to his estate.
* The corporation redeems all of Hank’s stock from his estate.
* The result is a total redemption and Hank’s estate has completely terminated its interest in the business.
* This distribution will not be subjected to dividend treatment for Sam because Hank’s estate has given up all control, share of future profits, and share of future assets in the event of a sale or liquidation of the corporation.

If the beneficiary of the estate owns stock (constructive ownership) and the corporation does not simultaneously redeem those shares owned by the beneficiary at the time it redeems the estate’s shares, then whatever the corporation redeemed from the estate is considered a dividend.

Also, under the “family/trust/corporation” attribution rule, an individual is considered to own the stock owned directly or indirectly by or for the individual’s spouse, children, grandchildren, and parents.
* Therefore, a close family member to an estate beneficiary who owns shares can make part or all of the redemption amount to be considered dividend distributions.
* This rule can be waived if the stockholder holds no interest after the redemption and agrees to not acquire interest for the next 10 years except through inheritance or by bequest.

52
Q

What is the purpose of trying to qualify a stock redemption as a Sec. 303 redemption?
* Avoid being taxed as capital gains.
* To extend control beyond family members.
* To break the chain of family attribution.
* Avoid being taxed as dividends.

A

Avoid being taxed as dividends.
* IRC Sec. 303 allows a corporation to make a distribution in redemption of a portion of the stock of a decedent that will not be taxed as a dividend to use to pay death taxes and other expenses.

53
Q

What are the Requirements of Section 303: Stock-Redemptions?

A

The IRC requirements for a Sec. 303 stock redemption are:
* The redeemed stock must be included in the decedent’s gross estate for federal estate tax purposes.
* The value for purposes of federal estate tax of all stock of the corporation that is included in determining the value of the decedent’s gross estate must be more than 35% of the excess of the value of the gross estate minus the sum allowable as a deduction under IRC Sec. 2053 that is estate expenses, indebtedness, and taxes, and Sec. 2054 that is losses. Gifts of all property made within 3 years of death are included in the gross estate before the percentage test is applied.
* Only an amount equal to the total of:
1. All estate taxes, inheritance, legacy, and succession taxes including generation-skipping transfer taxes and interest thereon imposed by reason of decedent’s death, and
2. Funeral and administration expenses whether or not claimed as a deduction on the federal estate tax return can be redeemed and receive favorable income tax treatment, that is, avoid dividend treatment.
* Any excess will be taxed under the rules of IRC Sec. 302. This means any balance may be taxed as a dividend to the seller, the executor or heir from whom the stock is being redeemed, or the balance may qualify for favorable tax treatment, that is, no realization of taxable gain due to the stepped-up basis of the stock upon the decedent’s death.
* A redemption under Sec. 303 will qualify for favorable tax treatment only to the extent that the interest of a shareholder whose stock is redeemed is reduced either directly or indirectly through a binding obligation to contribute toward the payment of the decedent’s administration expenses and death taxes.

54
Q

Diego’s gross estate is $2,250,000 & the administrative and funeral costs are $250,000. Calculate the value that the stock must exceed to qualify for Sec. 303 redemption.
* $2,000,000
* $350,000
* $700,000
* $1,000,000

A

$700,000

  • The value for purposes of federal estate tax of all stock of the corporation that is included in determining the value of the decedent’s gross estate must be more than 35% of the excess of the value of the gross estate minus the sum allowable as a deduction under IRC Sec. 2053 that is estate expenses, indebtedness, and taxes, and Sec. 2054 that is losses.

In Diego’s situation, the stock value is calculated as follows:
0.35 x ($2,250,000 - $250,000) = $700,000

55
Q

How is Section 303: Stock-Redemption Done?

A

In IRC Sec. 303 stock redemption, the corporation redeems stock from the party who receives it at the death of the decedent-stockholder.
* The amount of the shares to be redeemed will receive Sec. 303 treatment and not be treated as dividends.
* Usually, the recipient of the stock is the decedent shareholder’s personal representative, that is, the decedent’s executor or administrator.

Sometimes the seller is a direct heir, surviving spouse, or trustee of an irrevocable trust created by the decedent. The redemption is protected under Sec. 303 only if it is from a stockholder who is obligated to pay death taxes, funeral or administration expenses, or whose share of the decedent’s estate is reduced by these expenses.

You cannot redeem stock from any stockholder who has acquired his stock by purchase or gift if the donor was not the decedent. Also, Sec. 303 is not applicable where stock is redeemed from a stockholder who has acquired the stock from the executor in satisfaction of a specific monetary bequest.

The purchase of life insurance on the stockholder by the corporation guarantees the necessary cash on the insured’s death. The life insurance used to fund the Sec. 303 redemption should be a typical key person policy. The corporation should be the applicant, owner, premium payer, and beneficiary.

56
Q

Section 303 Redemption Example:

Assume Aaron, a widower who dies in 2017, owns 75% of a corporation. His son Joshua owns the remaining 25%. Aaron’s stock is valued at $1,200,000. His gross estate less allowable deductions under IRC Sec. 2053 and Sec. 2054 is $3,000,000. Assume he has minimal estate liquidity.

The family corporation is in the 37% federal income tax bracket. Aaron is in a combined federal and state 28% income tax bracket. The corporation would purchase $650,250 of life insurance on Aaron’s life. This assumes Aaron’s estate and inheritance taxes and other death-related expenses will approximate $650,250.

At Aaron’s death,
* his stock will pass to his estate.
* The corporation then receives the insurance proceeds on Aaron’s life.
* The corporation uses the life insurance proceeds to pay Aaron’s estate for stock qualifying for the Sec. 303 redemption.
* The estate transfers $650,250 worth of stock to the corporation.
* Aaron’s estate uses the cash to pay federal and state death taxes and administrative and funeral expenses.

A
57
Q

What are the Tax Implications of Section 303: Stock-Redemption?

A

The amount paid to the estate is not treated as a dividend distribution. It is treated as the exchange price for the stock and generally results in no gain being recognized at all by the estate. This happens if the basis for the stock has been stepped-up for being included in the shareholder’s estate.
* However, to the extent the price paid to the estate exceeds the estate’s basis, it will pay a tax on any such gain.

Under the step-up-in-basis rules, a Sec. 303 redemption may result in no adverse income tax consequences to the shareholder from whom the corporation made the redemption.

The favorable tax treatment occurs because the basis of the stock in the executor’s hands can be stepped up to the stock’s fair market value for federal estate tax purposes.

In some cases, the value per share paid by the corporation is greater than the value of the stock per share for federal estate tax purposes. When that happens, a capital gain results.

Sec. 303 redemptions are specifically exempt from attribution, that is, constructive ownership problems, which helps to make redemptions from family corporations possible without the threat of dividend treatment.

58
Q

Tax Implications of Section 303 Stock-Redemption Example:

A shareholder acquired his stock at $10 a share in 2022. When he passed away in 2023, his shares were worth $26 a share.
* The cost basis for the estate is stepped up from $10 to $26 per share.
* The price paid to the executor by the corporation to redeem the stock is generally equal to the fair market value for federal estate tax purposes.
* Thus, the amount realized on the sale is exactly equal to the seller’s basis in 2022 and there is no taxable gain for income tax purposes.

A

In some cases, the value per share paid by the corporation is greater than the value of the stock per share for federal estate tax purposes. When that happens, a capital gain results.

Sec. 303 redemptions are specifically exempt from attribution, that is, constructive ownership problems, which helps to make redemptions from family corporations possible without the threat of dividend treatment.

59
Q

What are the Implications in Community Property States?

A

In order to qualify or aggregate ownership interest in two or more corporations for purposes of meeting the more than 35% of adjusted gross estate test, there must be at least 20% in value of each corporation included in the decedent’s estate.
* In determining the 20% stock ownership by the decedent, half of the surviving spouse’s stock constituting community property may be included by treating it as if it had been included in determining the value of the gross estate of the decedent.

60
Q

Section 3 - Business Continuation Summary

The owner of a business must acknowledge the fact that one day he or she may be unable to run it anymore due to death, disability, or retirement. A business continuation agreement ensures the smooth continuation of the business beyond an owner’s involvement. The business may be passed on to family members, back to the company, other business associates, or an outside entity. A business continuation agreement can help map out how an owner’s interest is to be sold, who will purchase the interest, and how the sale would be funded.

In this lesson, we have covered the following:
* Defining business continuation: A business continuation agreement is for the disposition of a business interest in the event of the owner’s death or disability. It is also used in case of retirement or the owner’s withdrawal from the business. Business continuation can be used when a guaranteed market must be created or when it is necessary to peg the value of the business. The agreement can also be used if a partner is unable or unwilling to run the business with the deceased co-owner’s family or the business involves a high amount of financial risk for the family. It may be used if it is necessary to prevent the business from falling into the hands of outsiders, and where it is desirable to lend certainty to the disposition of a family closely-held business.

A
  • Stock-redemption agreements generally establish the value of the decedent’s interest in the business for federal estate tax purposes. A properly drawn-up corporate stock redemption agreement will ensure that the redemption will not be treated as a dividend distribution. Disability income and life insurance premiums used to fund the agreement are not deductible by the corporation. Premiums paid by the corporation are not taxable income to its shareholders.
  • Section 303 stock redemption is used when you want to keep control of a family corporation. It can also be used when the corporation’s stock is a major estate asset and a forced sale or liquidation of the business poses a threat. The amount paid to the estate is not treated as a dividend distribution but treated as the exchange price for the stock.
61
Q

Business continuation agreements take a number of forms. Which of the following are types of business continuation agreements? (Select all that apply)
* Cross-purchase agreement
* Entity plan
* Defined contribution plan
* Third party business buyout agreement
* Fractional share bequest

A

Cross-purchase agreement
Entity plan
Third party business buyout agreement
* The cross-purchase agreement is an agreement between the individual owners.
* The entity plan is an agreement between the business itself and the individual owners.
* The third party business buyout agreement is an agreement between the individual owners and a key person, family member, or outside individual.
* These are some forms that the business continuation agreement takes. Corporations commonly use stock redemption plans or shareholder cross-purchase plans.

62
Q

Ronald, the executor to the owner of Wells Inc. is in a meeting with his financial planner Margaret Plummer. He would like to know whether some of the stock can be redeemed as Sec. 303 stock redemption to pay for funeral expenses and estate taxes in the current year. Margaret explains the situations where he can use a Sec. 303 stock redemption. In which of the following situations can he use Sec. 303? (Select all that apply)
* When the corporation’s stock is a major estate asset and a forced sale or liquidation of the business in order to pay death taxes and other costs is a threat.
* Where a tax-favored withdrawal of funds from the corporation at the death of the stockholder would be useful.
* When redemption of IRC Sec. 303 stock is desirable.
* When the stock to be redeemed is not a part of the decedent’s gross estate for estate tax purposes.

A

When the corporation’s stock is a major estate asset and a forced sale or liquidation of the business in order to pay death taxes and other costs is a threat.
Where a tax-favored withdrawal of funds from the corporation at the death of the stockholder would be useful.
When redemption of IRC Sec. 303 stock is desirable.

  • Ronald can use the Sec. 303 redemption to take care of death taxes and other costs which could possibly cause liquidation.
  • The other situations where this can be used are when it would be useful to withdraw funds from the corporation on the death of the stockholder and when redemption is desirable.
63
Q

Life insurance proceeds and disability income used for paying for a stock redemption are not tax deductible to a corporation, but rather are corporate income tax free.
* False
* True

A

True
* Insurance proceeds and disability income used for paying for a stock redemption are considered tax free for corporate income.

64
Q

Section 4 - Intra-Family and Other Business Transfer Techniques

Intra-family and other business transfer techniques are various devices and strategies available to business owners to transfer their business interests to family members according to their objectives. Typically, business owners want to remove their business interests and future appreciation from their estate by selling or gifting to family members, while minimizing transfer taxes in the process.

To ensure that you have an understanding of intra-family business transfer techniques, the following topics will be covered in this lesson:
* Installment Note
* Self-Canceling Installment Note (SCIN)
* Private Annuity
* Gift- or Sale-Leaseback
* Family Limited Partnership (FLP) or Limited Liability Company (LLC)
* Intra-family Loan
* Bargain Sale

A

Upon completion of this lesson you should be able to:
* Define characteristics of installment notes,
* Explain when to use SCINs,
* Define characteristics of private annuities,
* Describe the uses of gift- or sale-leasebacks,
* Explain uses of FLPs and LLCs,
* Explain tax consequences of intra-family loans, and
* Define bargain sales.

65
Q

Describe using an Installment Note Describe as a intra-family sale technique

A

Installment notes are used primarily when business owners want to sell their business to family members to remove the business and any future appreciation from their estate and receive secured income payments from the sale.
* Since installment payments will be received over a period of time rather than as a lump sum, owners can defer their capital gains and recognize smaller gains each year when payments are received.

Family members who do not have sufficient funds to buy the business outright or make a down payment will find that an installment sale provides them with greater payment flexibility.
* The purchase price must be paid at some future time but a set sales price is not required, and payments can vary, be spread out over many years, and even be paid from future sales.
* The IRS requires that at least one payment must be made to the seller after the taxable year in which the sale occurs, but regular payments can begin and end whenever both parties choose.

Each payment received by the seller is comprised of a tax-free return of capital based on the seller’s adjusted basis, capital gains, and interest taxed at ordinary income rates.
* If parents charge too little interest on the sale, they run afoul of the IRS unstated interest rules, and must report income on imputed interest that should have been charged, but was not received.
* The seller must also report depreciation and investment tax credit recapture in the year of the sale, even if no payments are received.
* The buyer’s basis consists of the principal payments made and an interest deduction may be taken for the interest paid.
* If the family members who buy the business resell it within two years after the first payment is made, the seller must pay tax on the entire outstanding capital gain.

Installment sales freeze the value of the business in the owner’s estate if the owner dies during the installment sale term since the appreciation in the business has been transferred to the new owners.
* The present value of the outstanding installment payments is included in the seller’s gross estate, and the heirs are responsible for paying the entire income tax liability.

If the business owner outlives the installment sale term, then nothing is included in the gross estate other than any unconsumed principal and interest payments.

66
Q

Describe using a Self-Canceling Installment Note (SCIN) as a intra-family sale technique

A

A self-canceling installment note (SCIN) is indicated when a business owner wishes to sell his business to family members and receive income from the sale for a period of time, or until his death.
* A SCIN is structured as an installment sale with a provision that the note will partially or fully cancel automatically before the note matures or at the business owner’s death.
* The advantage of a SCIN is that any outstanding installment payments are not included in the business owner’s estate, unlike a typical installment sale.
* However, any unrealized capital gain must be recognized when the note is canceled or at the owner’s death.
* In the latter situation, it would be reported on the estate’s income tax return (Form 1041).

The buyer pays a premium for the business when using a SCIN since the actual payments received by the seller may be less than the current value of the business, since payments may end prematurely due to a lifetime cancellation or cancellation at the owner’s death.
* The premium may be a higher principal amount and/or a higher interest rate, and for cancellations at death, they are based on mortality tables.
* For example, a 65-year-old seller with a 20-year installment note has a mortality factor for the likelihood of dying before age 85. This mortality factor is multiplied by the amount of the note to arrive at the premium paid for his business.
* This extra premium means the seller will have a greater income tax liability from the sale of the business than he would have had using a regular installment sale.

A SCIN is better than an installment sale for a seller who is not expected to live long.
* It is also advantageous when the estate tax benefits from excluding the future installment payments are greater than the additional income tax the seller pays on the premium received since estate tax rates are higher than personal income tax rates.
* The business and future appreciation are removed from the seller’s estate and transferred to family members at a substantially reduced cost when using this transfer technique.

A lifetime cancellation of an installment note before the note matures results in an accelerated capital gains tax that must be recognized by the seller.
* In addition, the seller has made a gift to the buyer for the present value of the outstanding payments due.
* The gift tax liability can be reduced by the annual exclusion, but the seller might be subject to both income tax and gift tax in the year of cancellation.
* To avoid this, the seller may wish to make annual cancellations of the note to spread out the income tax and gift tax liability over time.
* The seller could also opt to cancel the payment in an amount that equals the annual exclusion each year, thus avoiding any gift tax liability on the annual cancellations.

67
Q

Describe using a Private Annuity as a intra-family sale technique

A

A private annuity is another intra-family sale technique used when a business or property owner wants to transfer his assets to family members and receive fixed income payments for life.
* In a single-life private annuity, a sale of the business or property is structured to permanently remove the assets from the owner’s estate, and not have the remaining payments included in the gross estate.

Private annuity payments are calculated as lifetime annuity payments, using US Treasury actuarial tables for the seller’s life expectancy and current IRC discount rates under Section 7520.
* Payments are unsecured promises to pay a monthly fixed payment to the seller for life, and are received by the seller as a tax-free return of adjusted basis, capital gains, and interest taxed as ordinary income.
* Once the seller’s basis in the property has been returned, all remaining payments are taxed as ordinary income.
* However, if the seller dies sooner than his actuarial life expectancy, the unrealized gain will be included on his estate’s income tax return, even though the remaining payments will not be included in his estate.

The buyer’s basis in the business is the amount of payments made to the seller.
* If a buyer were to die before the seller, the buyer’s estate must continue to make payments to the seller for the remainder of his life.
* If the seller outlives his actuarial life expectancy, the buyer must also continue to make payments until the seller’s death.
* The buyer cannot deduct the payments made to the seller on his personal income tax return.

A private annuity can also be structured as a joint and survivor annuity, where payments are made to one individual until their death, and then to the other annuitant until their death.
* Actuarial tables calculate a factor using two life expectancies based on the individuals’ current age and the prevailing IRC Section 7520 discount rate.
* If the annuitants are spouses, at the first spouse’s death, the present value of the future payments to the survivor is included in the decedent spouse’s estate but will receive a marital deduction equal to that amount.
* No payments are included in the second spouse’s estate at death.

68
Q

Describe using a Gift-Leaseback as a intra-family sale technique

A

Gift- or sale-leasebacks are techniques that can provide income to family members when the business owner does not have sufficient liquid assets available for this purpose. A business owner may have most of his wealth tied into tangible, illiquid business assets that cannot be sold to third parties to provide income since the assets are needed to run the business.

A sale or gift leaseback technique can structure a sale or gift of the business property to family members to provide them with:
* an income stream from lease payments and
* remove the business property from the owner’s estate.

In a gift-leaseback, the business owner may gift the land, building, and/or equipment used in the business directly to a family member, or irrevocably transfer the property into a trust with an independent trustee and name family members as beneficiaries.
* The business owner removes the value of this property and any future appreciation from his gross estate but the transfer is subject to a potential gift tax liability.
* Since the business owner still needs the transferred property to operate the business, he will lease back the business property from the recipient or the trust.
* The lease payments will provide the donee or trust beneficiaries with income payments that are taxed in their tax brackets if over age 18, which may be considerably lower than the business owner’s bracket, thus providing a tax advantage to the family.
* The business owner deducts the lease payments as an ordinary and necessary business expense and continues to use the property in the business.

69
Q

Describe using a Sale-Leaseback as a intra-family sale technique

A

A sale-leaseback works essentially the same way, except that the business property is sold to family members rather than gifted.
* The sale of the property must be based on the property’s appraised fair market value as an “arm’s length” transaction.
* The sale to family members can be structured as an installment sale, with the lease payments providing taxable income to satisfy installment payments to the owner.
* The owner and spouse can gift back an amount of installment payments equal to the annual exclusion amounts, to provide an income stream to the new property owners.
* The owner may consider selecting a property that has been fully depreciated, although depreciation is recaptured in the year of sale.
* The new owners can take depreciation deductions for the property acquired, and a deduction for expenses relating to the maintenance of the property.
* The value of the property is removed from the owner’s estate, the appreciation is passed on to family members, and the property is now outside the reach of the business owner’s creditors.

It is important that these intra-family techniques be set up properly to avoid problems with the IRS.
* The sale or gift of property must be irrevocable and based on fair market value, and a legally enforceable lease agreement should be in place that provides for reasonable lease payments.
* A sale-leaseback arrangement must also be established for a necessary business purpose, rather than simply used as a method of shifting income to other family members.

70
Q

Describe Family Limited Partnerships (FLP). What can & can’t be transferred?

A

A family limited partnership (FLP) is a legal, tax-paying entity established under state law, which is a partnership consisting entirely of family members.
* The FLP allows senior family members to transfer property to junior family members at significantly reduced transfer costs, to reduce the value of their estates.
* Senior family members are general partners (GPs) who can control the management of the partnership, control the flow of income to other family members, and receive a salary for services rendered.
* Children are limited partners who have ownership interest but no management control over the family enterprise.

A FLP must have a legitimate business purpose to exist.
* Parents may transfer their business or other investment property into the FLP in exchange for all equity interests.
* They may choose to receive 1% of general partnership units and 99% of limited partnership units.
* The initial transfer does not trigger an income tax, and any future income distributions are taxed to the partners who receive them.

The types of property that can be transferred into the FLP include:
* Stock of a closely held business but not voting shares. If voting shares are transferred and voted, the shares are included in the GP’s estate.
* Securities, if less than 80% of the value of the FLP property.
* Bonds.
* Real estate. Gain is recognized if the property has debt in excess of its income tax basis. Property located out of state avoids ancillary probate in the FLP.
* Life insurance policy on the GP’s life. Cash values are at the GP’s disposal. If the FLP owns the policy, partnership income is used to pay the premiums.

Properties that cannot be transferred into the FLP include:
* S-Corporation stock,
* a family residence, or
* qualified retirement assets.

71
Q

Describe using a FLP as a intra-family sale technique

A

General partners reduce the value of their estates by gifting limited partnership units to their children.
* Parents can gift all of their limited partnership units and keep a 1% general partnership interest to maintain control over the entire entity.
* If all limited partnership units are gifted, then at the general partner’s death only the general partnership interest is included in the gross estate, at its original appraised value.
* In addition, the deceased general partner may receive a lack of marketability and loss of control discounts for the value of the units included in the estate.

It’s clear that a FLP is better than an irrevocable trust for intra-family transfers.
* In a trust, if the grantor had control over the trust assets, the value of those assets would be included in the grantor’s estate at death after 2010.
* However in a FLP, the general partner can have complete control over all partnership interests, and possibly have less than 1% of the value of the general partner interest included in their gross estate after 2010.
* If the FLP owns a life insurance policy on the general partner’s life, then only 1% of the value is included in the general partner’s estate.

The value of the property held in a FLP is less than its value if held outside a FLP, because discounts can be taken for a lack of marketability and minority interests on transfers to limited partners.
* A valuation expert is needed to appraise the underlying value of the partnership interests, and determine the acceptable value of the combined discounts.
* The use of discounts leverages the parent’s annual exclusion gifts, since they can gift a greater amount of limited partnership units to their children, without incurring any gift tax.
* Spouses can also use gift splitting and their applicable credit amounts to avoid any gift tax liability on the transfers.
* More partnership interests can be transferred to limited partners over time using discounts and these gifting techniques.
* The limited partners will receive partnership interests, potential appreciation on their interests, and income distributions if stated in the FLP agreement.
* If income distributions are not required per the FLP agreement, then the limited partner has no present economic interest and annual exclusions cannot be used to offset gift taxes on transfers to limited partners.

FLPs have the additional advantage of providing some measure of creditor protection for limited partners.
* Creditors can only receive a “charging order” which is a court order requiring that all partnership distributions to a debtor be paid to the creditor until the debt is repaid.
* Since general partners control distributions, they can decide not to make any distributions to the debtor.
* The creditor then has no way of forcing a distribution or reaching the property held by the FLP.
* The creditor even has to pay a proportionate share of income taxes on the distribution that was not received.
* This deters creditors from filing charging orders since they would need to wait for general partners to either liquidate the partnership or make distributions to the debtor partner.

FLPs may also protect the property of limited partners who are going through a divorce, since most states do not award FLP property to a divorced spouse. The partnership agreement may prevent non-family ownership by giving limited partners the right to buy out another partner’s interest first.

Some of the disadvantages of a FLP are:
* Costs of having an attorney prepare the partnership agreement, appraisal costs, and annual tax preparation costs.
General partners are personally responsible for all debts.

72
Q

Describe using Limited Liability Company (LLC) as a intra-family sale technique

A

An alternative to a FLP is a Limited Liability Company (LLC). An LLC offers the tax advantages of a partnership (the pass-through of income and losses to individuals) and limited liability for family members.
* Management of the LLC can be controlled by certain family members who are actively involved in the business, but ownership can be given to other family members, which entitles them to receive distributions from profits.
* Gifts of non-voting membership interests to family members qualify for annual exclusions if they are deemed present interests.
* These transfers remove the annual exclusion amounts and future appreciation from the managing members’ estates.
* Management interests are not transferable without the consent of all members, and creditors can only obtain a lien against a member’s interest in the LLC, not against the business entity.

Practitioner Advice:
* The IRS is scrutinizing FLPs and LLCs, and has ruled against some taxpayers for not following proper formalities and for having too much control over distributions and liquidations.
* The IRS has penalized taxpayers for not giving family members ownership interests with any real, present, economic value, and has disallowed their annual exclusion transfers.
* It is very important that experienced attorneys draw up these agreements and perhaps consult with other financial advisors in the process, to discuss the tax and other financial planning ramifications to the family.

73
Q

Practitioner Advice:

Practitioner Advice:
* The IRS is scrutinizing FLPs and LLCs, and has ruled against some taxpayers for not following proper formalities and for having too much control over distributions and liquidations.
* The IRS has penalized taxpayers for not giving family members ownership interests with any real, present, economic value, and has disallowed their annual exclusion transfers.
* It is very important that experienced attorneys draw up these agreements and perhaps consult with other financial advisors in the process, to discuss the tax and other financial planning ramifications to the family.

A
74
Q

Describe using Intra-Family Loans as a intra-family sale technique

A

Personal loans made to family members are common and serve as viable intra-family techniques for transferring wealth to younger generations.
* Money or other assets used to fund these family loans will reduce the value of the lender’s gross estate and will remove any taxable appreciation from the assets transferred.

Family loans can be structured in many different ways.
* For example, a parent can loan cash to a grantor trust for the beneficiary’s use, in exchange for a promissory note. If the beneficiary invests the borrowed funds and earns a rate of return that exceeds the interest on the note, then the parent has transferred wealth to a family member without paying transfer taxes.
* However, any outstanding balance on the promissory note will be included in the grantor’s estate if the grantor dies before the note is paid.

Most family loans, however, are typically structured as lump-sum payments made directly to family members to help pay for first homes, cars, education expenses or new business start-up costs.
* There are clear advantages for making loans available to family members since interest rate charges and loan repayment terms are often more flexible and favorable than what banks can offer.
* Unfortunately, there are also financial and emotional risks involved, unfavorable tax consequences, and endless record-keeping responsibilities.
* To minimize these risks, loan agreements should be written documents with terms that specify the repayment amount, maturity date, interest charges, repayment schedules, and any late fee charges.

Parents will often make loans to their children with no intention of ever having them repaid. These loans are “forgiven” and the IRS views these loans as disguised gifts from the parent to the child.
* The loan amount is subject to gift taxes and the child would not report any income from the gift.
* A parent can avoid paying gift taxes by forgiving the loan payments and interest, or any skipped payments, up to the annual exclusion amounts each year.
* Of course, gift-splitting and the applicable credit could offset any taxable gifts on forgiven amounts that exceed the $17,000 (2023) annual exclusion.

Loans to third parties typically include repayment amounts with a prevailing market interest rate.
* The interest rate is determined by the applicable federal interest rate (AFR) issued monthly by the IRS.
* A loan payment from the borrower to the lender is a tax-free return of capital, while the interest is taxed at the lender’s ordinary income rate.

Family loans, however, are often gift loans which are interest-free loans or loans with below-market interest rates.
* Gift loans are usually recharacterized as arm’s length transactions so that the parent is deemed to have made a loan at the market rate, even though no actual interest has been charged.
* This forgone interest, the difference between the interest rate that should have been charged on the loan and the amount that is actually charged, generates a “phantom” interest payment instead. This phantom payment is considered a gift from the parent to the child and is subject to gift taxes and the annual exclusion.
* If the loan has a specified maturity date, the value of the gift is the difference between the amount loaned and the discounted present value of the note, as of the date of the loan.
* If the loan is a demand loan that can be repaid at any time, a gift is made on the last day of each year the loan remains outstanding and is based on the interest that should have been paid less any interest that the borrower actually paid.

Gift loans may generate income taxes as well.
* Parents must report taxable income for imputed interest that should have been paid by the child at the market interest rate, above any interest the child actually paid on the note.
* The child may take an income tax deduction for this “payment” for investment interest, such as for home mortgage interest, if they itemize, but the deduction does not apply to personal interest.

The IRS allows for exceptions to these rules.
* Intra-family gift loans are exempt from gift and income taxes if they are structured as interest-free or below-market rate loans that do not exceed $10,000 per person.
* This rule does not apply if the loan is used to purchase income producing assets.
* If the loan exceeds $10,000 but is less than $100,000, the imputed interest deemed paid by the child and received by the parent is not taxed if the child’s other net investment income is $1,000 or less.
* When the child’s net investment income exceeds $1,000 the tax is based solely on the imputed interest amount equal to the child’s net investment income.
* Note that parents will pay income taxes on all phantom interest on loans that exceed $100,000 and are used for any purpose.

75
Q

Practitioner Advice:

Practitioner Advice:
* When set up properly, person-to-person loans can be beneficial to both the borrowers and the lenders alike.
* However, the average default rate on interpersonal loans is 14% while the default rate for banks is only 1%.
* When intra-family loans are left undocumented or informally managed, these loans may run into repayment problems which strain relationships and create tax problems.

There are specialty loan administration companies, such as National Family Mortgage, which focus solely on facilitating loans between friends and family members. They provide a full range of loan services while protecting relationships between relatives and friends.

A
76
Q

Describe a Bargain Sale

A

A bargain sale is just what the name implies. It is a sale of property to family members, third parties, or charities for less than fair market value.
* The transaction is considered to be part sale and part gift.
* This removes the property from the seller’s estate except for any unconsumed proceeds that remain in the estate at death.

The difference between the fair market value price and the price paid for the property is the gifted portion of the sale.
* This is subject to gift taxes with annual exclusion offsets since the sale is a present interest transaction.
* Gifted amounts that exceed the annual exclusion will be brought back into the seller’s estate as an adjusted taxable gift.

A taxpayer may also have to recognize a taxable gain on a bargain sale for property sold to charity.

77
Q

Bargain Sale Taxation Example:

A donor sells property to a charity at the original purchase price rather than at the property’s fair market value. If the property is worth $10,000 and the donor paid $4,000 for the property more than 12 months ago, 4/10 of the property is the sale portion and 6/10 or $6,000 is the gift to charity.
* The gift is 60% of his cost basis of $4,000 or $2,400.
* The donor has a new cost basis of $1,600 to apply against the $4,000 he realized on the bargain sale, which gives him a long-term capital gain of $2,400 subject to capital gains tax.
* The gift of $6,000 saves the donor $2,220 in taxes if he is in the highest bracket of 37%.
* He has recovered his original investment of $4,000 plus saved $2,220 for a total recovery of $6,220 minus the capital gains tax paid.

A
78
Q

Section 4 - Intra-Family and Other Business Transfer Techniques Summary

Intra-family transfer techniques are available to business owners to freeze and transfer their business interests and property to family members, in ways that reduce gift and estate transfer taxes. Transfer techniques are selected based on the business owner’s objectives since available techniques can accomplish different objectives.

In this lesson we have covered the following:
* Installment Note is used to sell a business or other property to family members who cannot afford to pay a down payment or make payments at regular intervals. This is a secured promise to pay with flexible payments, which removes the business and the business appreciation from the owner’s estate.
* Self-Cancelling Installment Note is used when the business owner does not want the present value of the outstanding installment payments included in his estate at death.
* Private Annuity is a sale of property or a business to family members who make unsecured, fixed payments to the owner throughout the remainder of his life.

A
  • Gift or Sale Leaseback is used by business owners who do not have sufficient liquidity from their business to transfer funds to family members. Allows owner to gift or sale business property to children then lease it back, providing children with lease payments and the business owner with lease deductions.
  • Family Limited Partnership is a business entity that allows general partners the ability to reduce their estates, use discounts to leverage annual exclusion gifts, and maintain control of the business by gifting limited partnership shares to other family members.
  • Limited Liability Company is a business entity that allows voting members to reduce their estates, use discounts to leverage annual exclusion gifts, and maintain control of the business by gifting non-voting units to other family members.
  • Intra-family Loan is used to transfer funds to family members which reduces the lender’s estate and avoids transfer taxes if structured properly.
  • Bargain Sale is a sale for less than fair market value that is part sale and part gift.
79
Q

Ted Allard, age 72, owns 1/6 interest in a ski resort in Vermont, which is currently valued at $1.5 million. The ski resort continues to appreciate every year, and Ted’s estate is now worth $4.5 million. Ted wants to give his ski resort interest to his son Bob, a wealthy portfolio manager, to remove it from his estate without paying gift taxes. However, he needs the income it generates for the rest of his life. What is the best transfer technique to use?
* Preferred stock recapitalization
* Private Annuity
* Installment Sale
* Bargain Sale

A

Private Annuity
* A private annuity is the best transfer technique since the entire $1.5 million interest in the ski resort plus the appreciation will be removed from Ted’s estate. Payments are unsecured, but Bob is wealthy and can presumably make payments throughout Ted’s life.
* A preferred stock recapitalization is a gift of partial business interests that would be subject to gift tax.
* A bargain sale results in a gift tax for the difference between the sale price and the fair market value.
* An installment sale would cause the PV of any outstanding installment payments to be included in Ted’s estate at death.

80
Q

Carol O’Neil, age 61, has a profitable graphic art business valued at $800,000, and personal property worth $250,000. Carol wants to retire from her business, and needs $3,000 per month in retirement income. Her niece, a recent art school graduate, would like to buy Carol’s business, but she doesn’t have enough money for a down payment. What is the best transfer technique to use?
* Private Annuity
* Installment Sale
* Intentionally Defective Grantor Trust
* Sale Leaseback

A

Installment Sale

  • An installment sale is the best transfer technique to use since Carol can sell the business to her niece with no down payment, and receive secured income payments to meet her retirement needs. The PV of any outstanding installment payments will be included in her estate at death, but this will not trigger an estate tax liability.
  • A private annuity is based on unsecured payments, and Carol needs to receive the income or take back the business if needed.
  • A sale leaseback is a device used to provide cash to buyers.
  • IDGT is a sale of the business to a trust to permanently remove the property from a grantor’s estate, which is not an important tax consideration for Carol.
81
Q

The Pastinos own a popular bakery in their neighborhood. Their daughter, Maria, is a divorced mother of two children, struggling to make ends meet. The Pastinos would like to remove their building and land from their estate, along with the future appreciation. They would also like to give Maria $20,000 a year in additional support but they are “cash poor” since their wealth is tied into their business. What is the best transfer technique to use?
* Business Continuation Agreement
* Gift Leaseback
* Grantor Retained Income Trust
* Private Annuity

A

Gift Leaseback

  • A gift leaseback is the best transfer technique to use since a gift of the bakery building and land would remove these assets and appreciation from the Pastinos’ estate. Lease payments to Maria would give her $20,000 a year in additional support, and are deducted by the bakery.
  • A business continuation agreement guarantees the sale of a decedent’s business interest to partners or a corporation.
  • A GRAT is a split interest trust that provides income to a grantor for a period of time.
  • A private annuity would provide income to the Pastinos for life, instead of to their daughter.
82
Q

Module Summary

Proper business planning is very important for family business owners to ensure the business will continue beyond their involvement. Estate freeze rules help the owner of the business retain current value and control of a business while transferring future appreciation to younger generations. Businesses can use valuation discounts to reduce the value of business interests transferred to family members. A business continuation plan can ensure the interests of the owners are properly sold once he or she is unable to manage the business due to disability, retirement, death or withdrawal. Intra-family business transfer techniques are available to transfer business or other property to family members in ways that avoid or minimize transfer taxes.

The key concepts to remember are:
* Estate freeze rule helps the owner of property to freeze the present value of his estate. It also allows him to shift the future growth to his successors. The owner passes on only the additional benefits of the stocks or property to his successors. He can create common stock with nonvoting or limited voting rights and pass those shares on to family members. Chapter 14 of the IRC facilitates the valuation of the business under the freeze rule for lifetime transfers of partial business interests to family members.
* Valuation of closely-held businesses: Experienced appraisers are needed to determine the value of the business assets and the discounts that can be applied to the transfer of business units to family members. Discounts such as lack of marketability and minority discount can be combined to reduce the value of the business shares gifted to family members. These discounts leverage the donor’s annual exclusion and hasten the removal of business interests from the owner’s estate.

A
  • Business continuation: A business continuation agreement facilitates the disposition of a business interest in the event of the owner’s death or his inability to manage the business anymore. The agreements take a number of forms, such as partnership liquidation agreement, cross-purchase agreement, buyout agreement or a combination of all these. In a cross-purchase agreement, the other business associates purchase the decedent’s interest in the company. In stock redemption plans, the corporation will buy back the interest. Both the corporation and the other owners can fund their purchase with life insurance proceeds, disability insurance income, or debt. Sec. 303 stock redemption can be used to offset a decedent’s estate tax and funeral expenses. Whether or not the redemption amount is treated as a dividend distribution depends on the future control of the corporation and other tax code requirements.
  • Intra-family business transfer techniques are various devices and strategies available to business owners to transfer their business interests and property to family members, according to their objectives. Typically, business owners want to remove their business interests and future appreciation from their estate by selling or gifting business property to family members, while minimizing transfer taxes in the process. Transfer techniques such as installment sales, self-canceling installment notes, private annuities, sale or gift leasebacks, family limited partnerships and LLCs, loans to family members, and bargain sales can accomplish specific family objectives in addition to lowering transfer taxes for senior family members.
83
Q

Payments from a private annuity are __ ____??____ __.
* unsecured
* secured by collateral
* tax-free
* tax-deferred

A

unsecured

  • Private annuity payments are unsecured promises to pay a monthly fixed payment to the seller for life and are received by the seller as a tax-free return of adjusted basis, capital gains, and interest taxed as ordinary income.
84
Q

Each installment note payment received by the seller is comprised of which of the following? (Select all that apply)
* Capital gains
* Tax-free return of capital
* Interest taxed at ordinary income rates
* Additional premiums

A

Capital gains
Tax-free return of capital
Interest taxed at ordinary income rates

Each payment received by the seller is comprised of:
* a tax-free return of capital based on the seller’s adjusted basis,
* capital gains, and,
* interest taxed at ordinary income rates.

85
Q

The IRS Valuation Manual states that the fractional interest discount is generally based on which of the following factors:
* Use of land
* Access to financing
* Size of the fractional interest
* Number of owners
* Size of tract and likelihood of partition

A

Use of land
Access to financing
Size of the fractional interest
Number of owners
Size of tract and likelihood of partition

  • Though the IRS Valuation Manual states that the fractional interest discount is generally based on the cost of dividing the land, such as survey costs, court costs, and legal fees, the following other factors must be considered:
  • Size of the fractional interest
  • Number of owners
  • Size of tract and likelihood of partition
  • Use of land
  • Access to financing
86
Q

A __ ____??____ __ is a legal, tax-paying entity established under state law that allows senior family members to transfer property to junior family members at significantly reduced transfer costs, to reduce the value of their estates.
* Sale-Leaseback
* FLP
* SCIN
* Private Annuity

A

FLP

  • A family limited partnership (FLP) is a legal, tax-paying entity established under state law, which is a partnership consisting entirely of family members. The FLP allows senior family members to transfer property to junior family members at significantly reduced transfer costs, to reduce the value of their estates.
87
Q

Identify the two most common discounts available for family and closely-held businesses.
* Minority interest discount
* Blockage discount
* Key-person discount
* Lack of marketability discount

A

Minority interest discount
Lack of marketability discount

  • The two most common discounts available for family and closely-held businesses are:
  • Lack of marketability, and
  • A minority interest discount.
  • A lack of marketability discount is permitted for family businesses because investors are not interested in closely held stock or family limited partnership shares, and the cost of taking this stock public or selling it on an exchange to potential investors is very expensive.
  • A minority interest discount is allowed when transferring business interests to minority shareholders because these shareholders have no influence or control over business operations or management policy.
88
Q

Life insurance and disability income premiums used to fund the purchase of an owner’s interest are __ ____??____ __ to the corporation.
* not deductible
* tax-deferred
* deductible
* tax-exempt

A

not deductible

  • Life insurance and disability income premiums used to fund the purchase of an owner’s interest are not deductible to the corporation.
  • On the other hand, the death proceeds and disability income proceeds will be received by the insurance corporation on an income tax-free basis.
89
Q

Diana’s gross estate is $4,500,000 & the administrative and funeral costs are $350,000. Calculate the value that the stock must exceed to qualify for Sec. 303 redemption.
* $350,000
* $4,150,000
* $1,452,500
* $1,575,000

A

$1,452,500

  • The value for purposes of federal estate tax of all stock of the corporation that is included in determining the value of the decedent’s gross estate must be more than 35% of the excess of the value of the gross estate minus the sum allowable as a deduction under IRC Sec. 2053 that is estate expenses, indebtedness, and taxes, and Sec. 2054 that is losses.

In Diana’s situation, the stock value is calculated as follows:
0.35 x ($4,500,000 - $350,000) = $1,452,500

90
Q

When using a private annuity for intra-family transfers, once the seller’s basis in the property has been returned, all remaining payments are taxed as __ ____??____ __.
* capital gains
* ordinary income
* tax-free return of basis
* passive income

A

ordinary income

  • Once the private annuity seller’s basis in the property has been returned, all remaining payments are taxed as ordinary income.
91
Q

An item of income in respect of a decedent (IRD) may have which of the following tax liabilities: (Select all that apply)
* Self-employment tax
* Generation-skipping transfer tax
* Estate tax
* Income tax

A

Estate tax
Income tax

  • Since the recipient of the item of IRD must include the assets as taxable income, it is possible to have an item of IRD subject to two tax liabilities:
  • estate tax
  • income tax
92
Q

Intra-family gift loans are exempt from gift and income taxes if they are structured as interest-free or below-market-rate loans that do not exceed __ ____??____ __ per person.
* $1,000
* $15,000
* $10,000
* $100,000

A

$10,000

  • Intra-family gift loans are exempt from gift and income taxes if they are structured as interest-free or below-market-rate loans that do not exceed $10,000 per person.
93
Q

Which of the following intra-family transfer techniques should be implemented when a business owner:
wishes to sell their business to family members
receive income from the sale for a period of time, or until his death
partial or fully automatic cancelation at death
* Private Annuity
* Installment Note
* Intra-Family Loan
* SCIN

A

SCIN
* A self-canceling installment note (SCIN) is indicated when a business owner wishes to sell his business to family members and receive income from the sale for a period of time, or until his death.
* A SCIN is structured as an installment sale with a provision that the note will partially or fully cancel automatically before the note matures or at the business owner’s death.